Esercizi misti limiti 7

Esercizi misti sui Limiti

Home » Esercizi misti limiti 7

Richiamiamo di seguito solo i principali risultati che verranno utilizzati per la risoluzione degli esercizi. Per i richiami teorici più completi si rimanda alle dispense di teoria sui limiti notevoli , alla dispensa sui simboli di Landau e a quella sulle forme indeterminate.

Teorema 1. 

Siano f, g\colon A \subseteq \mathbb{R}\to \mathbb{R}, sia x_0 \in \mathbb{R} \cup \{\pm \infty\} un punto di accumulazione per A. Si assuma che

    \[\exists \lim\limits_{x \to x_0} f(x) =: \ell_1, \qquad \exists \lim\limits_{x \to x_0} g(x) =: \ell_2,\]

allora, ogni qualvolta l’espressione a destra non è un forma indeterminata, si ha:

    \[\begin{aligned} \exists \; \lim\limits_{x \to x_0}(f\pm g)(x) & =\ell_1 \pm \ell_2 \\ \exists \; \lim\limits_{x \to x_0}(f\cdot g)(x) & = \ell_1 \cdot \ell_2, \end{aligned}\]

Se x_0 è un punto di accumulazione per \{x \in A \colon g(x) \neq 0\}, allora si ha:

    \[\exists \; \lim\limits_{x \to x_0} \left( \dfrac{f}{g}\right)(x) = \dfrac{\ell_1}{\ell_2},\]

ogni qualvolta l’espressione a destra esiste e non è una forma indeterminata.

 

Teorema 2 – Teorema di sostituzione. 

Sia f\colon A \subseteq \mathbb{R}\to \mathbb{R} e sia x_0 \in \mathbb{R}\cup \{\pm \infty\}. Si assuma che

    \[\exists \lim\limits_{x \to x_0} f(x) = \ell \in \mathbb{R}\cup \{\pm \infty\}.\]

Sia I(\ell) un intorno di \ell e sia g \colon I(\ell) \to \mathbb{R} tale che

  1. se \ell \in \mathbb{R}, g è continua in \ell;
  2. se \ell = \pm \infty, allora esiste \lim\limits_{y \to \ell}g(y).

Allora,

    \[\lim\limits_{x \to x_0} g(f(x)) = \lim\limits_{y \to \ell}g(y).\]

 

 

Teorema 3 – Teorema di L’Hôpital. 

Siano A\subset \mathbb{R} e x_0\in \mathbb{R}\cup \{\pm\infty\} punto di accumulazione per A. Siano f,g:A\setminus \{x_0\}\rightarrow \mathbb{R} derivabili nel loro dominio e inoltre si supponga g^\prime(x)\neq0 \, \, \forall x \in I \setminus \{x_0\} . Se f,g sono entrambe infinitesime o infinite per x \rightarrow x_0 e se esiste il seguente limite

    \[\lim_{x \rightarrow x_0}\dfrac{f^\prime(x)}{g^\prime(x)}=\ell \in \mathbb{R}\cup \{\pm\infty\},\]

allora

    \[\lim_{x \rightarrow x_0}\dfrac{f(x)}{g(x)}=\ell.\]

 

 

Testi degli esercizi

Esercizio 7  (\bigstar\bigstar\bigstar\largewhitestar\largewhitestar). Calcolare, se esistono, i seguenti limiti:

    \[\begin{aligned} &1.\quad  \lim_{x \to +\infty}\left(\cos\frac{1}{x}\right)^{x^2};\\[10pt] & 2.\quad  \lim_{x \to 0} \dfrac{\sqrt[4]{1-4x^2+x^4}-1+x^2}{x^4};\\[10pt] &3.\quad  \lim_{x \to \frac{\pi}{4}} \dfrac{-\cos(2x) \; \sqrt{2} \cos(x+\frac{\pi}{4})}{(\ln(\tan x))^2};\\[10pt] &4.\quad  \lim_{x \to \frac{\pi}{2}}\left \vert  \tan x\right \vert ^{2x-\pi};\\[10pt] &5.\quad \lim_{x\to+\infty}x\left(\arctan x-\dfrac{\pi}{2}\right).\end{aligned}\]

 
Svolgimento.
 
1. Applichiamo il teorema 2 ponendo t=1/x, per cui se x\rightarrow +\infty allora t\rightarrow 0^+. Si ha dunque:

    \[\begin{aligned} 			\lim_{x \to +\infty}\left(\cos\frac{1}{x}\right)^{x^2} &= \lim_{t \to 0^+} \left(\cos t\right)^{\frac{1}{t^2}}\\ 			& =  \lim_{t \to 0^+} \left(1+(1-\cos t)\right)^{\frac{1}{t^2}}\\ 			& =  \lim_{t \to 0^+} \left(\left(1+(1-\cos t)\right)^{\frac{1}{1-\cos t}}\right)^{\frac{1-\cos t}{t^2}}\\ 			& =e^{-1/2}=\frac{1}{\sqrt{e}}. 		\end{aligned}\]

 
2. Eliminiamo la radice quarta dal numeratore, moltiplicando numeratore e denominatore prima per \dfrac{\sqrt[4]{1-4x^2+x^4}-(x^2-1)}{x^4} e successivamente per \sqrt{x^4-4x^2+1}+(x^2-1)^2. Così facendo, si ottiene:

    \[\begin{aligned} &\lim_{x \to 0}  \dfrac{\sqrt[4]{1-4x^2+x^4}+(x^2-1)}{x^4}= \\ & = \lim_{x \to 0} \dfrac{\sqrt{x^4-4x^2+1}-(x^2-1)^2}{x^4} \cdot \dfrac{1}{\sqrt[4]{1-4x^2+x^4}-(x^2-1)} \\ & = \lim_{x \to 0} \dfrac{x^4-4x^2+1-(x^2-1)^4}{x^4} \cdot \dfrac{1}{\sqrt[4]{1-4x^2+x^4}-(x^2-1)} \cdot \dfrac{1}{\sqrt{x^4-4x^2+1}+(x^2-1)^2} \\ & = \dfrac{1}{4} \; \lim_{x \to 0} \dfrac{x^4-4x^2+1-x^8+4x^6-6x^4+4x^2-1}{x^4} \\ & = \dfrac{1}{4} \;\lim_{x \to 0} \dfrac{-5x^4-x^8+4x^6}{x^4} = - \dfrac{5}{4} \end{aligned}\]

 
3. Posto y = x - \frac{\pi}{4} in virtù del teorema 2 si ha:

    \[\lim_{x \to \frac{\pi}{4}} \dfrac{-\cos(2x) \; \sqrt{2} \cos(x+\frac{\pi}{4})}{(\ln(\tan x))^2}=\lim_{y \to 0} \dfrac{-\cos\left(2y +\frac{\pi}{2}\right) \; \sqrt{2} \cos\left(y +\frac{\pi}{2}\right)}{(\ln\left(\tan \left(y +\frac{\pi}{4}\right)\right)^2}.\]

Si osserva che applicando gli archi associati si ha

    \[\begin{aligned} \cos\left(y +\dfrac{\pi}{2}\right) =-\sin y,  \qquad &\cos \left( 2y +\dfrac{\pi}{2}\right) = -\sin  \left( 2y \right) , \end{aligned}\]

e applicando le regole di somma e sottrazione per la tangente si ha

    \[\tan \left( x + \dfrac{\pi}{4}\right)=\dfrac{\tan x + \tan \dfrac{\pi}{4}}{1-\tan x \tan \frac{\pi}{4}}=\dfrac{\tan x +1}{1-\tan x },\]

da cui

    \[\ln\left( \tan \left( y + \dfrac{\pi}{4}\right)\right)=\ln \left( \dfrac{\tan y +1}{1-\tan y }\right) =\ln \left(1+\tan y \right)-\ln \left( 1 -\tan y \right) .\]

Tenendo conti di queste uguaglianze, possiamo riscrivere il limite come:

    \[\begin{aligned} &\lim_{y \to 0} \dfrac{-\cos\left(2y +\frac{\pi}{2}\right) \; \sqrt{2} \cos\left(y +\frac{\pi}{2}\right)}{(\ln\left(\tan \left(y +\frac{\pi}{4}\right)\right)^2}=\\ &=-\lim_{y \rightarrow 0}\dfrac{\sqrt{2}\sin y \sin \left(2y \right)}{\left(\ln\left(1+\tan y \right)- \ln \left(1-\tan y \right) \right)^2}\\ &=-\sqrt{2}\lim_{y \rightarrow 0}\dfrac{\sin y}{y}\cdot \dfrac{\sin \left(2y \right)}{2y}\cdot \dfrac{2y^2}{\left(\ln\left(1+\tan y\right)- \ln \left(1-\tan y \right) \right)^2}\\ &=-\sqrt{2}\lim_{y \rightarrow 0}\dfrac{\sin y}{y}\cdot \dfrac{\sin \left(2y \right)}{2y}\cdot \dfrac{2\frac{y^2}{\tan^2 y}\cdot \tan^2 y}{\left(\ln\left(1+\tan y\right)- \ln \left(1-\tan y \right) \right)^2}\\ &=-\sqrt{2}\lim_{y \rightarrow 0}\dfrac{\sin y}{y}\cdot \dfrac{\sin \left(2y \right)}{2y}\cdot \dfrac{2\frac{y^2}{\tan^2 y}}{\left(\frac{1}{\tan y}\ln\left(1+\tan y\right)- \frac{1}{\tan y}\ln \left(1-\tan y \right) \right)^2}\\ &= -\sqrt{2}\cdot \dfrac{2}{4}=-\dfrac{\sqrt{2}}{2}. \end{aligned}\]

 
4. Posto y=x-\frac{\pi}{2}, in virtù del teorema 2 si ha:

    \[\lim_{x \to \frac{\pi}{2}} \left \vert \tan x\right \vert ^{2x-\pi} = \lim_{y \to 0} \left \vert \tan  \left(y+\dfrac{\pi}{2}\right)\right \vert ^{2y}.\]

Notiamo che per gli archi associati si ha

    \[\tan \left(y+\dfrac{\pi}{2}\right)=-\dfrac{1}{\tan y}\]

e quindi

    \[\begin{aligned} \lim_{y \to 0} \left \vert \tan  \left(y+\dfrac{\pi}{2}\right)\right \vert ^{2y}=&\lim_{x \rightarrow 0 }\left \vert\ -\dfrac{1}{\tan y}\right \vert^{2y}\\=&\lim_{x \rightarrow 0 }\left \vert\ \dfrac{1}{\tan y}\right \vert^{2y}\\ =&\lim_{y \to 0} e^{2y \ln\left \vert  \frac{1}{\tan y}\right \vert}\\ =&\lim_{y \to 0} e^{2y \left(\ln 1 -\ln \left \vert\tan y \right \vert \right)}\\ =&\lim_{y \to 0} e^{-2y \ln\left\vert  \tan y\right \vert }\\ =&\lim_{y \rightarrow 0}e^{-2\frac{\ln \left \vert \tan y  \right \vert }{y^{-1}}}. \end{aligned}\]

Dal momento che

    \[\lim_{y\rightarrow 0}\dfrac{-2\ln \left \vert \tan y \right \vert}{y^{-1}}=\dfrac{+\infty}{+\infty}\]

possiamo applicare il teorema 3 e dunque:

    \[\begin{aligned} \lim_{y \to 0} \left \vert \tan  \left(y+\dfrac{\pi}{2}\right)\right \vert ^{2y}=&\lim_{y \rightarrow 0}e^{-2\frac{\ln \left \vert \tan y  \right \vert }{y^{-1}}}\\ =& \lim_{y \rightarrow 0}e^{\frac{\frac{1+\tan^2 y}{\tan y }\cdot }{-y^{-2}}}\\ =&\lim_{y \rightarrow 0}e^{-\frac{y}{\tan y}\cdot y \left( 1+\tan^2 y\right)}=e^{0}=1. \end{aligned}\]

 
5. Poiché il limite si presenta come una forma indeterminata [\infty/\infty], applicando il teorema 3 si ha:

    \[\begin{aligned} \lim_{x\to+\infty}x\left(\arctan x-\dfrac{\pi}{2}\right)&=\lim_{x\to+\infty}\dfrac{\arctan x -\dfrac{\pi}{2}}{x^{-1}}\\ &=-\lim_{x\to+\infty}\dfrac{x^2}{1+x^2}\\ &=-\lim_{x\to+\infty}\dfrac{x^2}{x^2\left( 1+\dfrac{1}{x^2}\right)}=-1. \end{aligned}\]

error: Il contenuto è protetto!!